LSAT and Law School Admissions Forum

Get expert LSAT preparation and law school admissions advice from PowerScore Test Preparation.

User avatar
 Dave Killoran
PowerScore Staff
  • PowerScore Staff
  • Posts: 5852
  • Joined: Mar 25, 2011
|
#84259
Complete Question Explanation
(The complete setup for this game can be found here: lsat/viewtopic.php?t=1698)

The correct answer choice is (A)

If Lisa is the driver of one of the cars, then J and H must be in the other car, with F as the driver. Which car the groups are in cannot be determined, and the placement of K is uncertain:

PT61_O2010 LG Explanations_Q3_diagram.png

K is the major uncertainty in this question, and as this is a Could Be True question, you should look for K in the answers. Not surprisingly, answer choice (A) features K, and is correct.
You do not have the required permissions to view the files attached to this post.

Get the most out of your LSAT Prep Plus subscription.

Analyze and track your performance with our Testing and Analytics Package.